You are on page 1of 25

© 2006 R. C. Hibbeler. Published by Pearson Education, Inc., Upper Saddle River, NJ. All rights reserved.

This material is protected under all copyright laws as they currently exist. No portion of this material may
be reproduced, in any form or by any means, without permission in writing from the publisher.

For the exclusive use of adopters of the Hibbeler series of books.

Problem 4-99

Replace the force at A by an equivalent force and couple moment at point P.

Given:

F := 375N

a := 2m

b := 4m

c := 2m

d := 1m

θ := 30deg

Solution :

F = 375 N

MP := F ⋅ cos ( θ ) ⋅ ( a + c) − F ⋅ sin ( θ ) ⋅ ( b − d)

MP = 737 N⋅ m
Problem 4-102

Replace the force system by an equivalent force and couple moment at point O.

Units Used :
3
kip := 10 lb

Given :

F1 := 430lb F2 := 260lb

a := 2ft e := 5ft

b := 8ft f := 12

c := 3ft g := 5

d := a θ := 60deg

Solution:

FRx := F 2⋅ ⎜
⎛ g ⎞ − F ⋅ sin ( θ ) FRx = −272.39lb
1
2 2
⎝ g + f ⎠

FRy := F 2⋅ ⎜
⎛ ⎞ − F ⋅ cos ( θ )
f
FRy = 25 lb
1
2 2
⎝ g +f ⎠

2 2
FR := FRx + FRy FR = 274 lb

⎛ FRy ⎞
θ 1 := atan ⎜ θ 1 = 5.24 deg
⎝ −FRx ⎠


Mo := F 1⋅ cos ( θ ) ⋅ ( a) + F1⋅ sin ( θ ) ⋅ ( b) + F2⋅ ⎜
f ⎞⋅ e
2 2
⎝ g +f ⎠

Mo = 4.61 kip⋅ ft
© 2006 R. C. Hibbeler. Published by Pearson Education, Inc., Upper Saddle River, NJ. All rights reserved.
This material is protected under all copyright laws as they currently exist. No portion of this material may
be reproduced, in any form or by any means, without permission in writing from the publisher.

For the exclusive use of adopters of the Hibbeler series of books.

Problem 4-125

Replace the force and couple-moment system by an equivalent resultant force and couple
moment at point O. Express the results in Cartesian vector form.

Units Used:

kN := 1000N

Given:

M = Mx⋅ i + My⋅ j + Mz⋅ k

F = F x⋅ i + Fy⋅ j + Fz⋅ k

Mx := −20kN⋅ m Fx := 8kN

My := −70kN⋅ m Fy := 6kN

Mz := 20kN⋅ m Fz := 8kN

a := 3m

b := 3m e := 5m

c := 4m f := 6m

d := 6m g := 5m

Solution:

⎛ Fx ⎞ ⎛ Mx ⎞ ⎛⎜ −f ⎞
⎜ ⎜
F := ⎜ Fy ⎟ M := ⎜ My ⎟ r := ⎜ e ⎟
⎜ ⎜ ⎜g
⎝ Fz ⎠ ⎝ Mz ⎠ ⎝ ⎠

⎛8 ⎞ ⎛ −10 ⎞
FR := F MR := M + r × F

FR = 6 kN

MR = 18 kN⋅ m
⎜ ⎜
⎝8 ⎠ ⎝ −56 ⎠
© 2006 R. C. Hibbeler. Published by Pearson Education, Inc., Upper Saddle River, NJ. All rights reserved.
This material is protected under all copyright laws as they currently exist. No portion of this material may
be reproduced, in any form or by any means, without permission in writing from the publisher.

For the exclusive use of adopters of the Hibbeler series of books.

Problem 4-127

Replace the force and couple-moment system by an equivalent resultant force and couple
moment at point Q. Express the results in Cartesian vector form.

Units Used:

kN := 1000N

Given:

M = Mx⋅ i + My⋅ j + Mz⋅ k

F = Fx⋅ i + Fy⋅ j + Fz⋅ k

Mx := −20kN⋅ m Fx := 8kN

My := −70kN⋅ m Fy := 6kN

Mz := 20kN⋅ m Fz := 8kN

a := 3m

b := 3m e := 5m

c := 4m f := 6m

d := 6m g := 5m

Solution:

⎛ Fx ⎞ ⎛ Mx ⎞ ⎛0⎞
⎜ ⎜ ⎜
F := ⎜ Fy ⎟ M := ⎜ My ⎟ r := e

⎜F ⎜M ⎝g⎠
⎝ z⎠ ⎝ z⎠

⎛8⎞ ⎛ −10 ⎞
FR := F MR := M + r × F

FR = 6 kN

MR = −30 kN⋅ m
⎜ ⎜
⎝8⎠ ⎝ −20 ⎠
© 2006 R. C. Hibbeler. Published by Pearson Education, Inc., Upper Saddle River, NJ. All rights reserved.
This material is protected under all copyright laws as they currently exist. No portion of this material may
be reproduced, in any form or by any means, without permission in writing from the publisher.

For the exclusive use of adopters of the Hibbeler series of books.

Problem 4-161

Determine the coordinate direction angles of F, which is applied to the end A of the pipe
assembly, so that the moment of F about O is zero.

Units Used:

Given:

F := 20lb

a := 8in c := 6in

d := 10in
b := 6in

Solution:

Require Mo = 0. This happens when force F is directed either towards or away from point O.

⎛⎜ c ⎞ ⎛ 0.3 ⎞
u = ⎜ 0.8
r
r := ⎜ a + b ⎟ u :=
⎜ d r ⎜
⎝ ⎠ ⎝ 0.5 ⎠

If the force points away from O, then

⎛⎜ α ⎞ ⎛⎜ α ⎞ ⎛ 70.8 ⎞
⎜ β ⎟ := acos ( u) ⎜ β ⎟ = ⎜ 39.8 deg
⎜γ ⎜γ ⎜
⎝ ⎠ ⎝ ⎠ ⎝ 56.7 ⎠

If the force points towards O, then

⎛⎜ α ⎞ ⎛⎜ α ⎞ ⎛ 109.2 ⎞
⎜ β ⎟ := acos ( −u) ⎜ β ⎟ = ⎜ 140.2 deg
⎜γ ⎜γ ⎜
⎝ ⎠ ⎝ ⎠ ⎝ 123.3 ⎠
© 2006 R. C. Hibbeler. Published by Pearson Education, Inc., Upper Saddle River, NJ. All rights reserved.
This material is protected under all copyright laws as they currently exist. No portion of this material may
be reproduced, in any form or by any means, without permission in writing from the publisher.

For the exclusive use of adopters of the Hibbeler series of books.

Problem 4-162

Determine the moment of the force F about point O. The force has coordinate direction angles
α, β, γ. Express the result as a Cartesian vector.

Given:
a := 8in
F := 20lb

α := 60deg b := 6in

β := 120deg c := 6in

γ := 45deg d := 10in

Solution:

⎛⎜ c ⎞ ⎛⎜ cos ( α ) ⎞ ⎛ 298.0 ⎞
r := ⎜ a + b ⎟ Fv := F ⋅ ⎜ cos ( β ) ⎟ M := r × F v M = ⎜ 15.1 lb⋅ in
⎜ d ⎜ cos ( γ ) ⎜
⎝ ⎠ ⎝ ⎠ ⎝ −200.0 ⎠
© 2006 R. C. Hibbeler. Published by Pearson Education, Inc., Upper Saddle River, NJ. All rights reserved.
This material is protected under all copyright laws as they currently exist. No portion of this material may
be reproduced, in any form or by any means, without permission in writing from the publisher.

For the exclusive use of adopters of the Hibbeler series of books.

Problem 4-166

Determine the resultant couple moment of the two couples that act on the assembly. Member
OB lies in the x-z plane.

Given :

F1 := 400N

F2 := 150N

a := 500mm

b := 600mm

c := 400mm

θ := 45deg

Solution:

⎛⎜ F1 ⎞ ⎛⎜ 0 ⎞
F1v := ⎜ 0 ⎟ F2v := ⎜ F2 ⎟

⎝0 ⎠ ⎝0 ⎠
⎛ b ⋅ cos ( θ ) ⎞ ⎛0 ⎞ ⎛ b ⋅ cos ( θ ) ⎞
r1B := ⎜ 0 r1A := ⎜ a r2 := ⎜ −c
⎜ ⎜ ⎜
⎝ −b ⋅ sin ( θ ) ⎠ ⎝0 ⎠ ⎝ −b ⋅ sin ( θ ) ⎠

⎛ 63.64

( )
M := r1B × F1v + r1A × −F1v + r2 × F2v M ⎜ −169.71
= N⋅m
⎜ 263.64
⎝ ⎠
© 2006 R. C. Hibbeler. Published by Pearson Education, Inc., Upper Saddle River, NJ. All rights reserved.
This material is protected under all copyright laws as they currently exist. No portion of this material may
be reproduced, in any form or by any means, without permission in writing from the publisher.

For the exclusive use of adopters of the Hibbeler series of books.

Problem 4-167

Replace the force F having acting at point A by an equivalent force and couple moment at point
C.

Given:
F := 50lb
a := 10ft
b := 20ft
c := 15ft
d := 10ft
e := 30ft

Solution:

⎛⎜ d ⎞
rAB := ⎜ c ⎟
⎜ −e
⎝ ⎠
rAB
Fv := F ⋅
rAB

⎛ 0 ⎞
rCA := ⎜ a + b

⎝ e ⎠
⎛ 14.3 ⎞
FR := Fv FR = ⎜ 21.4 lb

⎝ −42.9 ⎠
⎛ −1928.6 ⎞
MR := rCA × F v MR = ⎜ 428.6 lb⋅ ft

⎝ −428.6 ⎠
© 2006 R. C. Hibbeler. Published by Pearson Education, Inc., Upper Saddle River, NJ. All rights reserved.
This material is protected under all copyright laws as they currently exist. No portion of this material may
be reproduced, in any form or by any means, without permission in writing from the publisher.

For the exclusive use of adopters of the Hibbeler series of books.

Problem 4-169

The horizontal force F acts on the handle of the wrench. Determine the moment of this force
about point O. Specify the coordinate direction angles α, β, γ of the moment axis.

Given:

F := 30N
a := 50mm
b := 200mm
c := 10mm
θ := 45deg

Solution :

⎛ sin ( θ ) ⎞ ⎛⎜ −c ⎞
Fv := F ⎜ −cos ( θ ) rOA := ⎜ b ⎟
⎜ ⎜a
⎝ 0 ⎠ ⎝ ⎠

⎛ 1.06 ⎞
MO := rOA × F v MO = ⎜ 1.06 N⋅ m

⎝ −4.03 ⎠

⎛⎜ α ⎞ ⎛⎜ α ⎞ ⎛ 75.7 ⎞
⎛ MO ⎞
⎜ β ⎟ := acos ⎜ ⎜ β ⎟ = ⎜ 75.7 deg
⎜γ ⎜γ ⎜
⎝ ⎠ ⎝ MO ⎠ ⎝ ⎠ ⎝ 159.6 ⎠
© 2006 R. C. Hibbeler. Published by Pearson Education, Inc., Upper Saddle River, NJ. All rights reserved.
This material is protected under all copyright laws as they currently exist. No portion of this material may
be reproduced, in any form or by any means, without permission in writing from the publisher.

For the exclusive use of adopters of the Hibbeler series of books.

Problem 4-137

Replace the three forces acting on the plate by a wrench. Specify the magnitude of the force
and couple moment for the wrench and the point P(x, y) where its line of action intersects the
plate.

3
Units Used: kN := 10 N

Given :

FA := 500N a := 4m

FB := 800N b := 6m

FC := 300N

Solution:

⎛ FA ⎞

FR := ⎜ FC ⎟

⎝ FB ⎠

Guesses x := 1m y := 1m

M := 100N⋅ m

FR⎛x ⎞ ⎛ b ⎞ ⎛⎜ 0 ⎞ ⎛ 0 ⎞ ⎛⎜ 0 ⎞
⎜ ⎜a × F + ⎜a × 0
Given M⋅ + y × FR =
⎜ ⎜ ⎜ C⎟ ⎜ ⎜ ⎟
⎝ 0 ⎠ ⎝ 0 ⎠ ⎝ 0 ⎠ ⎜⎝ FB ⎠

FR
⎝0 ⎠

⎛⎜ M ⎞
⎛ x ⎞ ⎛ 1.163 ⎞
⎜ x ⎟ := Find ( M , x , y) M = 3.07 kN⋅ m ⎜ =⎜ m
⎜y ⎝ y ⎠ ⎝ 2.061 ⎠
⎝ ⎠
© 2006 R. C. Hibbeler. Published by Pearson Education, Inc., Upper Saddle River, NJ. All rights reserved.
This material is protected under all copyright laws as they currently exist. No portion of this material may
be reproduced, in any form or by any means, without permission in writing from the publisher.

For the exclusive use of adopters of the Hibbeler series of books.

Problem 4-138

Replace the three forces acting on the plate by a wrench. Specify the magnitude of the force and
couple moment for the wrench and the point P(y,z) where its line of action intersects the plate.

Units Used :

Given :

FA := 80lb a := 12ft

FB := 60lb b := 12ft

FC := 40lb

Solution:

⎛ −FC ⎞

FR := ⎜ −FB ⎟

⎝ −FA ⎠

Guesses y := 1ft z := 1ft M := 1lb⋅ ft

FR ⎛0 ⎞ ⎛ 0 ⎞ ⎛⎜ −FC ⎞ ⎛ 0 ⎞ ⎛⎜ 0 ⎞
⎜ ⎜a × ⎜
Given M⋅ + y × FR = ⎜ 0 ⎟ + a × ⎜ −FB ⎟
⎜ ⎜ ⎜
⎝ 0 ⎠ ⎜⎝ 0 ⎠ ⎝ b ⎠ ⎜⎝ 0 ⎠
FR
⎝z ⎠
⎛⎜ M ⎞
⎛ y ⎞ ⎛ 0.414 ⎞
⎜ y ⎟ := Find ( M , y , z) M = −624 lb⋅ ft ⎜ =⎜ ft
⎜z ⎝ z ⎠ ⎝ 8.69 ⎠
⎝ ⎠
c02.qxd 1/23/06 4:52 PM Page 29

Article 2/3 Rectangular Components 29

Sample Problem 2/1 y


F2 = 500 N F1 = 600 N
The forces F1, F2, and F3, all of which act on point A of the bracket, are
specified in three different ways. Determine the x and y scalar components of
each of the three forces. 3 A 35°
4
0.1 m
x
Solution. The scalar components of F1, from Fig. a, are
0.2 m

F1x  600 cos 35  491 N Ans. 0.3 m

F1y  600 sin 35  344 N Ans. F3 = 800 N


B
The scalar components of F2, from Fig. b, are 0.4 m

4
F2x  500(5)  400 N Ans.
3
F2y  500(5)  300 N Ans. F1 = 600 N
F1 y
Note that the angle which orients F2 to the x-axis is never calculated. The cosine F3 x
A
and sine of the angle are available by inspection of the 3-4-5 triangle. Also note 35°

F3
that the x scalar component of F2 is negative by inspection. A F1 x

=8
The scalar components of F3 can be obtained by first computing the angle 
(a) α

00
of Fig. c.
0.4 m

N
  tan1 0.2
0.4
 26.6 F2 = 500 N
F2 y
F3 y

3
0.2 m B
 Then F3x  F3 sin   800 sin 26.6  358 N Ans. 4
F2 x A (c)
F3y  F3 cos   800 cos 26.6  716 N Ans.
(b)

Alternatively, the scalar components of F3 can be obtained by writing F3 as


a magnitude times a unit vector nAB in the direction of the line segment AB. Helpful Hints
Thus,
 You should carefully examine the
geometry of each component deter-
 
l
AB 0.2i  0.4j
 F3  F3nAB  F3   800 mination problem and not rely on
AB (0.2)2  (0.4)2 the blind use of such formulas as
 800 [0.447i  0.894j] Fx  F cos  and Fy  F sin .

 358i  716j N  A unit vector can be formed by di-


viding any vector, such as the geo-
The required scalar components are then l
metric position vector AB , by its
length or magnitude. Here we use
F3x  358 N Ans.
the overarrow to denote the vector
F3y  716 N Ans. which runs from A to B and the
overbar to determine the distance
which agree with our previous results. between A and B.
c02.qxd 1/23/06 4:52 PM Page 30

30 Chapter 2 Force Systems

Sample Problem 2/2 B


P = 800 lb
lb
Combine the two forces P and T, which act on the fixed structure at B, into 0
60 y
a single equivalent force R. =
T
6′ x

Graphical solution. The parallelogram for the vector addition of forces T and A α C 60° D
 P is constructed as shown in Fig. a. The scale used here is 1 in.  800 lb; a scale
of 1 in.  200 lb would be more suitable for regular-size paper and would give 3′
greater accuracy. Note that the angle a must be determined prior to construction
of the parallelogram. From the given figure

BD 6 sin 60 B 800 lb


tan     0.866   40.9 P
AD 3  6 cos 60 θ
lb
6 00
Measurement of the length R and direction  of the resultant force R yields the
α
approximate results T R

R  525 lb   49 Ans. (a)

Geometric solution. The triangle for the vector addition of T and P is shown Helpful Hints
 in Fig. b. The angle  is calculated as above. The law of cosines gives  Note the repositioning of P to per-
mit parallelogram addition at B.
R2  (600)2  (800)2  2(600)(800) cos 40.9  274,300

R  524 lb Ans.
800 lb
B P
From the law of sines, we may determine the angle  which orients R. Thus, θ α
R 600 lb
600 524 α
 sin   0.750   48.6 Ans.
sin  sin 40.9 T
(b)

Algebraic solution. By using the x-y coordinate system on the given figure,
we may write
 Note the repositioning of F so as to
Rx  ΣFx  800  600 cos 40.9  346 lb preserve the correct line of action of
the resultant R.
Ry  ΣFy  600 sin 40.9  393 lb

The magnitude and dipection of the resultant force R as shown in Fig. c are then
y

R  Rx2  Ry2  (346)2  (393)2  524 lb Ans.


Rx = 346 lb
Ry  393 B x
 tan1  tan 1  48.6 Ans. θ
Rx  346 Ry = – 393 lb

The resultant R may also be written in vector notation as R


(c)
R  Rxi  Ry j  346i  393j lb Ans.
c02.qxd 1/23/06 4:52 PM Page 31

Article 2/3 Rectangular Components 31

Sample Problem 2/3 y

The 500-N force F is applied to the vertical pole as shown. (1) Write F in j
terms of the unit vectors i and j and identify both its vector and scalar compo-
nents. (2) Determine the scalar components of the force vector F along the j′ y′
x- and y-axes. (3) Determine the scalar components of F along the x- and y-axes. A 30°
x
i

F = 500 N
Solution. Part (1). From Fig. a we may write F as 30°

F  (F cos )i  (F sin )j i′

 (500 cos 60)i  (500 sin 60)j x′

 (250i  433j) N Ans. y y′

The scalar components are Fx  250 N and Fy  433 N. The vector compo- Fx
A x A j′
nents are Fx  250i N and Fy  433j N. θ = 60°
Part (2). From Fig. b we may write F as F  500i N, so that the required F
scalar components are Fy F
i′
Fx  500 N Fy  0 Ans.

Part (3). The components of F in the x- and y-directions are nonrectan- (a) (b) x′
gular and are obtained by completing the parallelogram as shown in Fig. c. The
y′ Fx
magnitudes of the components may be calculated by the law of sines. Thus,
x
60° 30°
Fx 500 90°
  Fx  1000 N Fy′
90°
sin 90 sin 30 30° 60°
Fy F = 500 N
500
 Fy  866 N (c)
sin 60 sin 30

The required scalar components are then Helpful Hint


 Obtain Fx and Fy graphically and
Fx  1000 N Fy  866 N Ans.
compare your results with the calcu-
lated values.

a
Sample Problem 2/4 F1 = 100 N

Forces F1 and F2 act on the bracket as shown. Determine the projection Fb C


of their resultant R onto the b-axis. 30°
20°

F2 = 80 N b

Solution. The parallelogram addition of F1 and F2 is shown in the figure.


Using the law of cosines gives us a
F1
R2  (80)2  (100)2  2(80)(100) cos 130 R  163.4 N
0N
10 R
The figure also shows the orthogonal projection Fb of R onto the b-axis. Its C 50°
length is 80 N

Fb  80  100 cos 50  144.3 N Ans. F2 50°


Fb
Note that the components of a vector are in general not equal to the projec-
tions of the vector onto the same axes. If the a-axis had been perpendicular to b
the b-axis, then the projections and components of R would have been equal.
c02.qxd 1/23/06 4:52 PM Page 41

Article 2/4 Moment 41

Sample Problem 2/5 2m


A
Calculate the magnitude of the moment about the base point O of the 600-N
force in five different ways. 40°

4m 600 N

Solution. (I) The moment arm to the 600-N force is

O
d  4 cos 40  2 sin 40  4.35 m
2m
 By M  Fd the moment is clockwise and has the magnitude
40°
MO  600(4.35)  2610 N  m Ans.
4m
600 N
(II) Replace the force by its rectangular components at A 40° d

F1  600 cos 40  460 N, F2  600 sin 40  386 N O

By Varignon’s theorem, the moment becomes 2m F1 = 600 cos 40°

 MO  460(4)  386(2)  2610 N  m Ans.


4m
F2 = 600 sin 40°
(III) By the principle of transmissibility, move the 600-N force along its
line of action to point B, which eliminates the moment of the component F2. The
moment arm of F1 becomes
O
d1  4  2 tan 40  5.68 m B
F1 y

and the moment is F2


A
x
MO  460(5.68)  2610 N  m Ans.
d1 r F
 (IV) Moving the force to point C eliminates the moment of the component
F1. The moment arm of F2 becomes
C
F1
d2  2  4 cot 40  6.77 m O d2

and the moment is F2

MO  386(6.77)  2610 N  m Ans.


Helpful Hints
(V) By the vector expression for a moment, and by using the coordinate  The required geometry here and in
system indicated on the figure together with the procedures for evaluating cross similar problems should not cause dif-
products, we have ficulty if the sketch is carefully drawn.
 This procedure is frequently the
 MO  r  F  (2i  4j)  600(i cos 40  j sin 40) shortest approach.
 2610k N  m  The fact that points B and C are not
on the body proper should not cause
The minus sign indicates that the vector is in the negative z-direction. The mag- concern, as the mathematical calcula-
nitude of the vector expression is tion of the moment of a force does not
require that the force be on the body.
MO  2610 N  m Ans.  Alternative choices for the position
vector r are r  d1 j  5.68j m and
r  d2i  6.77i m.
c02.qxd 1/23/06 4:52 PM Page 42

42 Chapter 2 Force Systems

Sample Problem 2/6 B


T
The trap door OA is raised by the cable AB, which passes over the small fric- 0.3 m
tionless guide pulleys at B. The tension everywhere in the ca`le is T, and this ten- 0.4 m A
sion applied at A causes a moment MO about the hinge at O. Plot the quantity MO/T 0.5 m
as a function of the door elevation angle  over the range 0    90 and note min- O
θ
imum and maximum values. What is the physical significance of this ratio?

Solution. We begin by constructing a figure which shows the tension force T y


acting directly on the door, which is shown in an arbitrary angular position . It
should be clear that the direction of T will vary as  varies. In order to deal with B
rAB
this variation, we write a unit vector nAB which “aims” T: T
rAB rOB  rOA rOB A
 nAB  r  rAB d
AB rOA

Using the x-y coordinates of our figure, we can write O θ


x
 rOB  0.4j m and rOA  0.5(cos i  sin j) m
So Helpful Hints
rAB  rOB  rOA  0.4j  (0.5)(cos i  sin j)  Recall that any unit vector can be
 0.5 cos i  (0.4  0.5 sin )j m written as a vector divided by its
magnitude. In this case the vector in
and
the numerator is a position vector.
rAB  (0.5 cos )2  (0.4  0.5 sin )2
0.5
 0.41  0.4 sin  m
0.4
The desired unit vector is
0.3
rAB 0.5 cos i  (0.4  0.5 sin )j MO
nAB  r  ——– ,
T
m
AB 0.41  0.4 sin  0.2

Our tension vector can now be written as 0.1

T  TnAB  T 0.5 cos0.41


i  (0.4  0.5 sin )j
 0.4 sin 
 0
0 10 20 30 40 50 60 70 80 90
θ , deg
 The moment of T about point O, as a vector, is MO  rOB  T, where rOB  0.4j m, or
 Recall that any vector may be writ-
MO  0.4j  T 0.5 cos0.41
i  (0.4  0.5 sin )j
 0.4 sin 
 ten as a magnitude times an “aim-
ing” unit vector.
0.2T cos 
 k
0.41  0.4 sin   In the expression M  r  F, the po-
The magnitude of MO is sition vector r runs from the mo-
ment center to any point on the line
0.2T cos 
MO  of action of F. Here, rOB is more con-
0.41  0.4 sin  venient than rOA.
and the requested ratio is
MO 0.2 cos 
 Ans.
T 0.41  0.4 sin 
which is plotted in the accompanying graph. The expression MO/T is the moment
arm d (in meters) which runs from O to the line of action of T. It has a maximum
value of 0.4 m at   53.1 (at which point T is horizontal) and a minimum value of
0 at   90 (at which point T is vertical). The expression is valid even if T varies.
This sample problem treats moments in two-dimensional force systems, and
it also points out the advantages of carrying out a solution for an arbitrary posi-
tion, so that behavior over a range of positions can be examined.
c02.qxd 1/23/06 4:52 PM Page 52

52 Chapter 2 Force Systems

Sample Problem 2/7


M
The rigid structural member is subjected to a couple consisting of the two
100-N forces. Replace this couple by an equivalent couple consisting of the two 40 P
θ
forces P and P, each of which has a magnitude of 400 N. Determine the proper θ
angle . –P

100
Solution. The original couple is counterclockwise when the plane of the forces
is viewed from above, and its magnitude is
100
[M  Fd] M  100(0.1)  10 N  m

The forces P and P produce a counterclockwise couple


60
100
M  400(0.040) cos 
100 N

 Equating the two expressions gives 100 N

10  (400)(0.040) cos  Dimensions in millimeters

10
  cos1 16  51.3 Ans. P = 400 N
θ
Helpful Hint
40 mm

 Since the two equal couples are parallel free vectors, the only dimensions
which are relevant are those which give the perpendicular distances between θ
the forces of the couples. P = 400 N

Sample Problem 2/8


80 lb
Replace the horizontal 80-lb force acting on the lever by an equivalent sys-
tem consisting of a force at O and a couple.
9″

60°
O
Solution. We apply two equal and opposite 80-lb forces at O and identify the
counterclockwise couple

[M  Fd] M  80(9 sin 60)  624 lb-in. Ans. 80 lb 80 lb

 Thus, the original force is equivalent to the 80-lb force at O and the 624-lb-in.
couple as shown in the third of the three equivalent figures.
≡ ≡
Helpful Hint
O O
 The reverse of this problem is often encountered, namely, the replacement O
of a force and a couple by a single force. Proceeding in reverse is the same as 80 lb 80 lb 80 lb
624 lb-in.
replacing the couple by two forces, one of which is equal and opposite to the
80-lb force at O. The moment arm to the second force would be M/F 
624/80  7.79 in., which is 9 sin 60, thus determining the line of action of
the single resultant force of 80 lb.
c02.qxd 1/23/06 4:52 PM Page 60

60 Chapter 2 Force Systems

Sample Problem 2/9 y


2m
Determine the resultant of the four forces and one couple which act on the 60 N 5m
plate shown. 50 N

45°
Solution. Point O is selected as a convenient reference point for the force–couple 140 N·m
2m
system which is to represent the given system.
80 N
[Rx  ΣFx] Rx  40  80 cos 30  60 cos 45  66.9 N 2m
40 N O 30° x
[Ry  ΣFy] Ry  50  80 sin 30  60 cos 45  132.4 N 1m

[R  Rx2  Ry2] R  (66.9)2  (132.4)2  148.3 N Ans.


y

  tan 
Ry 132.4
1
  tan1  63.2 Ans.
Rx 66.9 R = 148.3 N
 [MO  Σ(Fd)] MO  140  50(5)  60 cos 45(4)  60 sin 45(7)
(a)
θ = 63.2°
 237 N  m |MO| =
237 N·m x
O
The force–couple system consisting of R and MO is shown in Fig. a.
We now determine the final line of action of R such that R alone represents
the original system.
R = 148.3 N
[Rd  MO] 148.3d  237 d  1.600 m Ans.
Hence, the resultant R may be applied at any point on the line which makes a (b) 1.600 m 63.2°
63.2 angle with the x-axis and is tangent at point A to a circle of 1.600-m radius A
with center O, as shown in part b of the figure. We apply the equation Rd  MO in x
O
an absolute-value sense (ignoring any sign of MO) and let the physics of the situa- B
tion, as depicted in Fig. a, dictate the final placement of R. Had MO been counter-
clockwise, the correct line of action of R would have been the tangent at point B.
The resultant R may also be located by determining its intercept distance b 132.4 x – 66.9 y =
to point C on the x-axis, Fig. c. With Rx and Ry acting through point C, only Ry –237 y
(c)
exerts a moment about O so that
C
x
237 O
Ry b  MO and b  1.792 m
132.4 R b
Alternatively, the y-intercept could have been obtained by noting that the mo-
ment about O would be due to Rx only. Helpful Hints
A more formal approach in determining the final line of action of R is to use
 We note that the choice of point O as
the vector expression
a moment center eliminates any mo-
r  R  MO ments due to the two forces which
pass through O. Had the clockwise
where r  xi  yj is a position vector running from point O to any point on the sign convention been adopted, MO
line of action of R. Substituting the vector expressions for r, R, and MO and car- would have been 237 N  m, with
rying out the cross product result in the plus sign indicating a sense
(xi  yj)  (66.9i  132.4j)  237k which agrees with the sign conven-
tion. Either sign convention, of
(132.4x  66.9y)k  237k course, leads to the conclusion of a
clockwise moment MO.
Thus, the desired line of action, Fig. c, is given by
 Note that the vector approach
132.4x  66.9y  237 yields sign information automati-
cally, whereas the scalar approach
 By setting y  0, we obtain x  1.792 m, which agrees with our earlier calcula- is more physically oriented. You
tion of the distance b. should master both methods.
c02.qxd 1/23/06 4:52 PM Page 69

Article 2/7 Rectangular Components 69

Sample Problem 2/10 F = 100 N


z
A force F with a magnitude of 100 N is applied at the origin O of the axes
4m
x-y-z as shown. The line of action of F passes through a point A whose coordi- A
nates are 3 m, 4 m, and 5 m. Determine (a) the x, y, and z scalar components of
F, (b) the projection Fxy of F on the x-y plane, and (c) the projection FOB of F
along the line OB. y B
5m
2m
6m
Solution. Part (a). We begin by writing the force vector F as its magnitude
F times a unit vector nOA. O 3m 6m

 
l
OA 3i  4j  5k
F  FnOA  F  100 x
OA 32  42  52
z
 100[0.424i  0.566j  0.707k]
F
 42.4i  56.6j  70.7k N

The desired scalar components are thus Fz


y
 Fx  42.4 N Fy  56.6 N Fz  70.7 N Ans.

Part (b). The cosine of the angle xy between F and the x-y plane is Fy

32  42 θ xy Fxy = 70.7 N


cos xy   0.707
32  42  52
O
so that Fxy  F cos xy  100(0.707)  70.7 N Ans. Fx

Part (c). The unit vector nOB along OB is x


l
OB 6i  6j  2k
nOB    0.688i  0.688j  0.229k
OB 62  62  22 F

The scalar projection of F on OB is

 FOB  F  nOB  (42.4i  56.6h 70.7k)  (0.688i  0.688j  0.229k)


z
 (42.4)(0.688)  (56.6)(0.688)  (70.7)(0.229)
nOB
 84.4N Ans. y
FOB = 84.4 N
If we wish to express the projection as a vector, we write

O
FOB  F  nOBnOB
x
 84.4(0.688i  0.688j  0.229k)

 58.1i  58.1j  19.35k N Helpful Hints


 In this example all scalar components
are positive. Be prepared for the case
where a direction cosine, and hence
the scalar component, are negative.
 The dot product automatically finds
the projection or scalar component
of F along line OB as shown.
c02.qxd 1/23/06 4:52 PM Page 78

78 Chapter 2 Force Systems

Sample Problem 2/11 z

Determine the moment of force F about point O (a) by inspection and (b) by b
the formal cross-product definition MO  r  F. a

Solution. (a) Because F is parallel to the y-axis, F has no moment about that c O
axis. It should be clear that the moment arm from the x-axis to the line of action y
of F is c and that the moment of F about the x-axis is negative. Similarly, the
moment arm from the z-axis to the line of action of F is a and the moment of F
about the z-axis is positive. So we have x

MO  cFi  aFk  F(ci  ak) Ans. z

(b) Formally, b
a

 MO  r  F  (ai  ck)  Fj  aFk  cFi F

 F(ci  ak) Ans. r


c O
y
Helpful Hint
 Again we stress that r runs from the moment center to the line of action of F.
Another permissible, but less convenient, position vector is r  ai  bj  ck. x

z
Sample Problem 2/12 1.6 m

The turnbuckle is tightened until the tension in cable AB is 2.4 kN. Deter-
A
mine the moment about point O of the cable force acting on point A and the
magnitude of this moment.

2m

Solution. We begin by writing the described force as a vector. O


y
x
T  TnAB  2.4 0.8  1.5  2 
0.8i  1.5j  2k
2 2 2
1.5 m
B
0.8 m

 0.731i  1.371j  1.829k kN z

1.6 m
The moment of this force about point O is
A
 MO  rOA  T  (1.6i  2k)  (0.731i  1.371j  1.829k)

 2.74i  4.39j  2.19k kN  m Ans. rOA

This vector has a magnitude 2m T

O
MO  2.742  4.392  2.192  5.62 kN  m Ans.
y
x
Helpful Hint 1.5 m
0.8 m
 The student should verify by inspection the signs of the moment components. B
c02.qxd 1/23/06 4:52 PM Page 79

Article 2/8 Moment and Couple 79

Sample Problem 2/13 y

A tension T of magnitude 10 kN is applied to the cable attached to the top A


of the rigid mast and secured to the ground at B. Determine the moment Mz of T A
about the z-axis passing through the base O.

15 m
Solution (a). The required moment may be obtained by finding the compo- T = 10 kN
nent along the z-axis of the moment MO of T about point O. The vector MO is
normal to the plane defined by T and point O, as shown in the accompanying fig-
O
ure. In the use of Eq. 2/14 to find MO, the vector r is any vector from point O to
 the line of action of T. The simplest choice is the vector from O to A, which is x
written as r  15j m. The vector expression for T is z 12 m 9m
B
T  TnAB  10 (12)  (15)  (9) 
12i  15j  9k
2 2 2
Helpful Hints

 10(0.566i  0.707j  0.424k) kN  We could also use the vector from O


From Eq. 2/14, to B for r and obtain the same result,
but using vector OA is simpler.
[MO  r  F] MO  15j  10(0.566i  0.707j  0.424k)
 It is always helpful to accompany your
 150(0.566k  0.424i) kN  m vector operations with a sketch of the
The value Mz of the desired moment is the scalar component of MO in the vectors so as to retain a clear picture
z-direction or Mz  MO  k. Therefore, of the geometry of the problem.

Mz  150(0.566k  0.424i)  k  84.9 kN  m Ans.  Sketch the x-y view of the problem
and show d.
 The minus sign indicates that the vector Mz is in the negative z-direction. Ex-
pressed as a vector, the moment is Mz  84.9k kN  m. y

A
Solution (b). The force of magnitude T is resolved into components Tz and Txy
in the x-y plane. Since Tz is parallel to the z-axis, it can exert no moment about
 this axis. The moment Mz is, then, due only to Txy and is Mz  Txyd, where d is r Mz
the perpendicular distance from Txy to O. The cosine of the angle between T and T Mo
Txy is 152  122 / 152  122  92  0.906, and therefore,
O
Txy  10(0.906)  9.06 kN
x
The moment arm d equals OA multiplied by the sine of the angle between Txy z
and OA, or B

12
d  15  9.37 m
122  152 y

Hence, the moment of T about the z-axis has the magnitude Tz A Tx

Mz  9.06(9.37)  84.9 kN  m Ans.

and is clockwise when viewed in the x-y plane. Ty


T Txy 15 m

Solution (c). The component Txy is further resolved into its components Tx and Ty. O
It is clear that Ty exerts no moment about the z-axis since it passes through it, so
that the required moment is due to Tx alone. The direction cosine of T with respect x
to the x-axis is 12/92  122  152  0.566 so that Tx  10(0.566)  5.66 kN. Thus, z
12 m 9m
B
Mz  5.66(15)  84.9 kN  m Ans.
c02.qxd 1/23/06 4:52 PM Page 80

80 Chapter 2 Force Systems

Sample Problem 2/14 30 N


30 N
Determine the magnitude and direction of the couple M which will replace 60°
the two given couples and still produce the same external effect on the block. mm 60°
Specify the two forces F and F, applied in the two faces of the block parallel to 100 mm 60
x
mm y
the y-z plane, which may replace the four given forces. The 30-N forces act paral- 40
lel to the y-z plane.
25 N
50 mm
Solution. The couple due to the 30-N forces has the magnitude M1  30(0.06) 
1.80 N •m. The direction of M1 is normal to the plane defined by the two forces,
and the sense, shown in the figure, is established by the right-hand convention. 25 N
The couple due to the 25-N forces has the magnitude M2  25(0.10)  2.50 N  m
z
with the direction and sense shown in the same figure. The two couple vectors
combine to give the components M2 = 2.5 N·m

My  1.80 sin 60  1.559 N  m M


x y
Mz  2.50  1.80 cos 60  1.600 N  m θ
–F
 Thus, M  (1.559)2  (1.600)2  2.23 N  m Ans. θ
y θ
1.559
with   tan1 1.600  tan1 0.974  44.3 Ans. 60° F
M1 = 1.8 N·m
The forces F and F lie in a plane normal to the couple M, and their mo-
z
ment arm as seen from the right-hand figure is 100 mm. Thus, each force has the z
magnitude
Helpful Hint

[M = Fd]
2.23
F  0.10  22.3 N Ans.  Bear in mind that the couple vectors
are free vectors and therefore have
and the direction   44.3. no unique lines of action.

Sample Problem 2/15 z

A force of 40 lb is applied at A to the handle of the control lever which is at- 40 lb


tached to the fixed shaft OB. In determining the effect of the force on the shaft 8″
at a cross section such as that at O, we may replace the force by an equivalent 2″
force at O and a couple. Describe this couple as a vector M. 3″ A
O
Solution. The couple may be expressed in vector notation as M  r  F, y
l
where r  OA  8j  5k in. and F  40i lb. Thus,
x
B
M  (8j  5k)  (40i)  200j  320k lb-in.
z
Alternatively we see that moving the 40-lb force through a distance d  (40 lb)
52  82  9.43 in. to a parallel position through O requires the addition of a M
couple M whose magnitude is 40 lb
θ A
M  Fd  40(9.43)  377 lb-in. Ans. d
θ 5″
The couple vector is perpendicular to the plane in which the force is shifted, and O
its sense is that of the moment of the given force about O. The direction of M in 8″
x
the y-z plane is given by y

5
  tan1 8  32.0 Ans.
c02.qxd 1/23/06 4:52 PM Page 91

Article 2/9 Resultants 91

Sample Problem 2/16 z

Determine the resultant of the force and couple system which acts on the 700 lb-in.
rectangular solid. 50 lb
1000 lb-in.

80 lb
Solution. We choose point O as a convenient reference point for the initial 50 lb
step of reducing the given forces to a force–couple system. The resultant force is 960
lb-in.
 R  ΣF  (80  80)i  (100  100)j  (50  50)k  0 lb
O 12″
The sum of the moments about O is 80 lb

 MO  [50(16)  700]i  [80(12)  960] j  [100(10)  1000]k lb-in. x 16″


10″ 100 lb
 100i lb-in.

Hence, the resultant consists of a couple, which of course may be applied at any 100 lb
point on the body or the body extended. y

Helpful Hints
 Since the force summation is zero, we conclude that the resultant, if it exists,
must be a couple.

 The moments associated with the force pairs are easily obtained by using the
M  Fd rule and assigning the unit-vector direction by inspection. In many
three-dimensional problems, this may be simpler than the M  r  F approach.

Sample Problem 2/17 50 N

Determine the resultant of the system of parallel forces which act on the 0.5 m
plate. Solve with a vector approach. z
x 0.5 m

O 0.35 m
Solution. Transfer of all forces to point O results in the force–couple system

R  ΣF  (200  500  300  50)j  350j N 500 N 0.35 m


MO  [50(0.35)  300(0.35)]i  [50(0.50)  200(0.50)]k y 200 N
 87.5i  125k N  m 300 N
The placement of R so that it alone represents the above force–couple system is
determined by the principle of moments in vector form z
x
r  R  MO x
z
(xi  yj  zk)  350j  87.5i  125k r O
R
350xk  350zi  87.5i  125k
From the one vector equation we may obtain the two scalar equations R
MO
y
350x  125 and 350z  87.5

Hence, x  0.357 m and z  0.250 m are the coordinates through which the
Helpful Hint
line of action of R must pass. The value of y may, of course, be any value, as
 permitted by the principle of transmissibility. Thus, as expected, the variable y  You should also carry out a scalar
drops out of the above vector analysis. solution to this problem.
c02.qxd 1/23/06 4:52 PM Page 92

92 Chapter 2 Force Systems

Sample Problem 2/18 x


700 N
500 N
Replace the two forces and the negative wrench by a single force R applied
at A and the corresponding couple M. 25 N· m
60°
40°
z B
30 80 mm
Solution. The resultant force has the components mm
A 50 mm
100
mm 45°
[Rx  ΣFx] Rx  500 sin 40  700 sin 60  928 N 60 mm
600 N
[Ry  ΣFy] Ry  600  500 cos 40 cos 45  871 N 40 mm
120 mm
y
[Rz  ΣFz] Rz  700 cos 60  500 cos 40 sin 45  621 N

Thus, R  928i  871j  621k N


R
M
and R (928)2  (871)2  (621)2  1416 N Ans.

The couple to be added as a result of moving the 500-N force is


A
 [M  r  F] M500  (0.08i  0.12j  0.05k)  500(i sin 40
 j cos 40 cos 45  k cos 40 sin 45)

where r is the vector from A to B.

The term-by-term, or determinant, expansion gives Helpful Hints

M500  18.95i  5.59j  16.90k N  m  Suggestion: Check the cross-product


results by evaluating the moments
about A of the components of the
 The moment of the 600-N force about A is written by inspection of its x- and z-
components, which gives 500-N force directly from the sketch.

M600  (600)(0.060)i  (600)(0.040)k  For the 600-N and 700-N forces it is


easier to obtain the components of
 36.0i  24.0k N  m their moments about the coordinate
directions through A by inspection
The moment of the 700-N force about A is easily obtained from the moments of of the figure than it is to set up the
the x- and z-components of the force. The result becomes cross-product relations.

M700  (700 cos 60)(0.030)i  [(700 sin 60)(0.060)  The 25-N  m couple vector of the
  (700 cos 60)(0.100)] j  (700 sin 60)(0.030)k wrench points in the direction oppo-
site to that of the 500-N force, and
 10.5i  71.4 j  18.19k N  m
we must resolve it into its x-, y-, and
z-components to be added to the
 Also, the couple of the given wrench may be written other couple-vector components.

M  25.0(i sin 40  j cos 40 cos 45  k cos 40 sin 45)
 Although the resultant couple vec-
 16.07i 13.54 j  13.54k N  m tor M in the sketch of the resultants
is shown through A, we recognize
Therefore, the resultant couple on adding together the i-, j-, and k-terms of the that a couple vector is a free vector
four M’s is and therefore has no specified line
of action.
 M  49.4i  90.5j  24.6k N  m

and M  (49.4)2  (90.5)2  (24.6)2  106.0 N  m Ans.


c02.qxd 1/23/06 4:52 PM Page 93

Article 2/9 Resultants 93

Sample Problem 2/19 z


4″
Determine the wrench resultant of the three forces acting on the bracket.
Calculate the coordinates of the point P in the x-y plane through which the resul- 3″
tant force of the wrench acts. Also find the magnitude of the couple M of the 40 lb y
wrench.
5″
x
20 lb
P
y
Solution. The direction cosines of the couple M of the wrench must be the
 same as those of the resultant force R, assuming that the wrench is positive. The 40 lb
resultant force is x

R  20i  40j  40k lb R  (20)2  (40)2  (40)2  60 lb z


4″
and its direction cosines are

y R
cos x  20/60  1/3 cos y  40/60  2/3 cos z  40/60  2/3 3″
x=
The moment of the wrench couple must equal the sum of the moments of 3i
n.
the given forces about point P through which R passes. The moments about P of
the three forces are P
5″
y = 2 in.
(M)Rx  20yk lb-in.
M x
(M)Ry  40(3)i  40xk lb-in.

(M)Rz  40(4  y)i  40(5  x)j lb-in. Helpful Hint

and the total moment is  We assume initially that the wrench


is positive. If M turns out to be neg-
M  (40  40y)i  (200  40x)j  (40x  20y)k lb-in. ative, then the direction of the cou-
ple vector is opposite to that of the
The direction cosines of M are resultant force.

cos x  (40  40y)/M

cos y  (200  40x)/M

cos z  (40x  20y)/M

where M is the magnitude of M. Equating the direction cosines of R and M gives

M
40  40y 
3

2M
200 40x 
3

2M
40x 20y 
3

Solution of the three equations gives

M  120 lb-in. x  3 in. y  2 in. Ans.

We see that M turned out to be negative, which means that the couple vector is
pointing in the direction opposite to R, which makes the wrench negative.

You might also like